Proving $f_nto f$a.e implies $f_nto f$ almost uniformly [duplicate]












1












$begingroup$



This question already has an answer here:




  • Omitting the hypotheses of finiteness of the measure in Egorov theorem

    3 answers





Exercise: Let $(f)_{ninmathbb{N}}$ be a sequence of functions such that $f_nto f$a.e(almost everyehere) and there exists $g$ integrable such that $|f_n|leqslant g$a.e
for all $ninmathbb{N}$. Prove that $f_nto f$ almost uniformly.




I think I can apply the following theorem:




Ergoroff Theorem:
Consider $Einmathscr{F}$(sigma-algebra), and $EinOmega$ defined on a measure space $(Omega,mathscr{F},mu)$. Suppose $mu(E)<infty$, and ${f_n}$ is a sequence of measurable functions on $Etomathbb{R}$ which are finite almost everywhere and converge almost everywhere to a function $f:Etomathbb{R}$ which is also finite almost everywhere. Then $f_nto f$ almost uniformly in $E$.




I now that $f_nto f$ a.e so $lim_{ntoinfty}f_n(x)=f(x)forall xin E$, But to apply Ergoroff theorem I need to prove that $mu(E)<infty$ or $mu(Omega)<infty$.
I know by the Dominated convergence theorem that $lim_{ntoinfty}int |f_n-f| dmu=0$ but I cannot see how shall I prove from there that $mu(E)$ or $mu(Omega)$ are limited.



Question:



Can someone provide me any help?



Thanks in advance!



Note:$f_n$ does not necessarily converge to $f$ uniformly. So the question is not a duplicate.










share|cite|improve this question











$endgroup$



marked as duplicate by Xander Henderson, mrtaurho, zz20s, Cesareo, metamorphy Jan 14 at 9:47


This question has been asked before and already has an answer. If those answers do not fully address your question, please ask a new question.


















  • $begingroup$
    If $lambda>0$ then $mu({x:g(x)>lambda})<infty$.
    $endgroup$
    – David C. Ullrich
    Jan 13 at 13:02










  • $begingroup$
    @Jakobian My question is not a duplicate. If you read carefully the exercise you find out $f_n$ does not necessarily converge to $f$ uniformly. However the answer you provide assumes $f_n$ to converge uniformly so it is not answering this question.
    $endgroup$
    – Pedro Gomes
    Jan 13 at 15:13










  • $begingroup$
    @PedroGomes no, it doesn't. It's exactly the answer to your question
    $endgroup$
    – Jakobian
    Jan 13 at 15:20












  • $begingroup$
    @DavidC.Ullrich In order to apply Ergoroff I need to prove the measure of the domain where the function converges is finite. It is already assumed in the question when $f_nto f$ a.e that $mu({x:g(x)>lambda})=0$.
    $endgroup$
    – Pedro Gomes
    Jan 13 at 15:21








  • 1




    $begingroup$
    How do I know that you are pointing me in the right direction if you do not prove you are? Does it not sound like an authority fallacy?
    $endgroup$
    – Pedro Gomes
    Jan 13 at 15:32
















1












$begingroup$



This question already has an answer here:




  • Omitting the hypotheses of finiteness of the measure in Egorov theorem

    3 answers





Exercise: Let $(f)_{ninmathbb{N}}$ be a sequence of functions such that $f_nto f$a.e(almost everyehere) and there exists $g$ integrable such that $|f_n|leqslant g$a.e
for all $ninmathbb{N}$. Prove that $f_nto f$ almost uniformly.




I think I can apply the following theorem:




Ergoroff Theorem:
Consider $Einmathscr{F}$(sigma-algebra), and $EinOmega$ defined on a measure space $(Omega,mathscr{F},mu)$. Suppose $mu(E)<infty$, and ${f_n}$ is a sequence of measurable functions on $Etomathbb{R}$ which are finite almost everywhere and converge almost everywhere to a function $f:Etomathbb{R}$ which is also finite almost everywhere. Then $f_nto f$ almost uniformly in $E$.




I now that $f_nto f$ a.e so $lim_{ntoinfty}f_n(x)=f(x)forall xin E$, But to apply Ergoroff theorem I need to prove that $mu(E)<infty$ or $mu(Omega)<infty$.
I know by the Dominated convergence theorem that $lim_{ntoinfty}int |f_n-f| dmu=0$ but I cannot see how shall I prove from there that $mu(E)$ or $mu(Omega)$ are limited.



Question:



Can someone provide me any help?



Thanks in advance!



Note:$f_n$ does not necessarily converge to $f$ uniformly. So the question is not a duplicate.










share|cite|improve this question











$endgroup$



marked as duplicate by Xander Henderson, mrtaurho, zz20s, Cesareo, metamorphy Jan 14 at 9:47


This question has been asked before and already has an answer. If those answers do not fully address your question, please ask a new question.


















  • $begingroup$
    If $lambda>0$ then $mu({x:g(x)>lambda})<infty$.
    $endgroup$
    – David C. Ullrich
    Jan 13 at 13:02










  • $begingroup$
    @Jakobian My question is not a duplicate. If you read carefully the exercise you find out $f_n$ does not necessarily converge to $f$ uniformly. However the answer you provide assumes $f_n$ to converge uniformly so it is not answering this question.
    $endgroup$
    – Pedro Gomes
    Jan 13 at 15:13










  • $begingroup$
    @PedroGomes no, it doesn't. It's exactly the answer to your question
    $endgroup$
    – Jakobian
    Jan 13 at 15:20












  • $begingroup$
    @DavidC.Ullrich In order to apply Ergoroff I need to prove the measure of the domain where the function converges is finite. It is already assumed in the question when $f_nto f$ a.e that $mu({x:g(x)>lambda})=0$.
    $endgroup$
    – Pedro Gomes
    Jan 13 at 15:21








  • 1




    $begingroup$
    How do I know that you are pointing me in the right direction if you do not prove you are? Does it not sound like an authority fallacy?
    $endgroup$
    – Pedro Gomes
    Jan 13 at 15:32














1












1








1





$begingroup$



This question already has an answer here:




  • Omitting the hypotheses of finiteness of the measure in Egorov theorem

    3 answers





Exercise: Let $(f)_{ninmathbb{N}}$ be a sequence of functions such that $f_nto f$a.e(almost everyehere) and there exists $g$ integrable such that $|f_n|leqslant g$a.e
for all $ninmathbb{N}$. Prove that $f_nto f$ almost uniformly.




I think I can apply the following theorem:




Ergoroff Theorem:
Consider $Einmathscr{F}$(sigma-algebra), and $EinOmega$ defined on a measure space $(Omega,mathscr{F},mu)$. Suppose $mu(E)<infty$, and ${f_n}$ is a sequence of measurable functions on $Etomathbb{R}$ which are finite almost everywhere and converge almost everywhere to a function $f:Etomathbb{R}$ which is also finite almost everywhere. Then $f_nto f$ almost uniformly in $E$.




I now that $f_nto f$ a.e so $lim_{ntoinfty}f_n(x)=f(x)forall xin E$, But to apply Ergoroff theorem I need to prove that $mu(E)<infty$ or $mu(Omega)<infty$.
I know by the Dominated convergence theorem that $lim_{ntoinfty}int |f_n-f| dmu=0$ but I cannot see how shall I prove from there that $mu(E)$ or $mu(Omega)$ are limited.



Question:



Can someone provide me any help?



Thanks in advance!



Note:$f_n$ does not necessarily converge to $f$ uniformly. So the question is not a duplicate.










share|cite|improve this question











$endgroup$





This question already has an answer here:




  • Omitting the hypotheses of finiteness of the measure in Egorov theorem

    3 answers





Exercise: Let $(f)_{ninmathbb{N}}$ be a sequence of functions such that $f_nto f$a.e(almost everyehere) and there exists $g$ integrable such that $|f_n|leqslant g$a.e
for all $ninmathbb{N}$. Prove that $f_nto f$ almost uniformly.




I think I can apply the following theorem:




Ergoroff Theorem:
Consider $Einmathscr{F}$(sigma-algebra), and $EinOmega$ defined on a measure space $(Omega,mathscr{F},mu)$. Suppose $mu(E)<infty$, and ${f_n}$ is a sequence of measurable functions on $Etomathbb{R}$ which are finite almost everywhere and converge almost everywhere to a function $f:Etomathbb{R}$ which is also finite almost everywhere. Then $f_nto f$ almost uniformly in $E$.




I now that $f_nto f$ a.e so $lim_{ntoinfty}f_n(x)=f(x)forall xin E$, But to apply Ergoroff theorem I need to prove that $mu(E)<infty$ or $mu(Omega)<infty$.
I know by the Dominated convergence theorem that $lim_{ntoinfty}int |f_n-f| dmu=0$ but I cannot see how shall I prove from there that $mu(E)$ or $mu(Omega)$ are limited.



Question:



Can someone provide me any help?



Thanks in advance!



Note:$f_n$ does not necessarily converge to $f$ uniformly. So the question is not a duplicate.





This question already has an answer here:




  • Omitting the hypotheses of finiteness of the measure in Egorov theorem

    3 answers








functional-analysis measure-theory proof-writing






share|cite|improve this question















share|cite|improve this question













share|cite|improve this question




share|cite|improve this question








edited Jan 13 at 15:14







Pedro Gomes

















asked Jan 13 at 12:27









Pedro GomesPedro Gomes

1,8262721




1,8262721




marked as duplicate by Xander Henderson, mrtaurho, zz20s, Cesareo, metamorphy Jan 14 at 9:47


This question has been asked before and already has an answer. If those answers do not fully address your question, please ask a new question.









marked as duplicate by Xander Henderson, mrtaurho, zz20s, Cesareo, metamorphy Jan 14 at 9:47


This question has been asked before and already has an answer. If those answers do not fully address your question, please ask a new question.














  • $begingroup$
    If $lambda>0$ then $mu({x:g(x)>lambda})<infty$.
    $endgroup$
    – David C. Ullrich
    Jan 13 at 13:02










  • $begingroup$
    @Jakobian My question is not a duplicate. If you read carefully the exercise you find out $f_n$ does not necessarily converge to $f$ uniformly. However the answer you provide assumes $f_n$ to converge uniformly so it is not answering this question.
    $endgroup$
    – Pedro Gomes
    Jan 13 at 15:13










  • $begingroup$
    @PedroGomes no, it doesn't. It's exactly the answer to your question
    $endgroup$
    – Jakobian
    Jan 13 at 15:20












  • $begingroup$
    @DavidC.Ullrich In order to apply Ergoroff I need to prove the measure of the domain where the function converges is finite. It is already assumed in the question when $f_nto f$ a.e that $mu({x:g(x)>lambda})=0$.
    $endgroup$
    – Pedro Gomes
    Jan 13 at 15:21








  • 1




    $begingroup$
    How do I know that you are pointing me in the right direction if you do not prove you are? Does it not sound like an authority fallacy?
    $endgroup$
    – Pedro Gomes
    Jan 13 at 15:32


















  • $begingroup$
    If $lambda>0$ then $mu({x:g(x)>lambda})<infty$.
    $endgroup$
    – David C. Ullrich
    Jan 13 at 13:02










  • $begingroup$
    @Jakobian My question is not a duplicate. If you read carefully the exercise you find out $f_n$ does not necessarily converge to $f$ uniformly. However the answer you provide assumes $f_n$ to converge uniformly so it is not answering this question.
    $endgroup$
    – Pedro Gomes
    Jan 13 at 15:13










  • $begingroup$
    @PedroGomes no, it doesn't. It's exactly the answer to your question
    $endgroup$
    – Jakobian
    Jan 13 at 15:20












  • $begingroup$
    @DavidC.Ullrich In order to apply Ergoroff I need to prove the measure of the domain where the function converges is finite. It is already assumed in the question when $f_nto f$ a.e that $mu({x:g(x)>lambda})=0$.
    $endgroup$
    – Pedro Gomes
    Jan 13 at 15:21








  • 1




    $begingroup$
    How do I know that you are pointing me in the right direction if you do not prove you are? Does it not sound like an authority fallacy?
    $endgroup$
    – Pedro Gomes
    Jan 13 at 15:32
















$begingroup$
If $lambda>0$ then $mu({x:g(x)>lambda})<infty$.
$endgroup$
– David C. Ullrich
Jan 13 at 13:02




$begingroup$
If $lambda>0$ then $mu({x:g(x)>lambda})<infty$.
$endgroup$
– David C. Ullrich
Jan 13 at 13:02












$begingroup$
@Jakobian My question is not a duplicate. If you read carefully the exercise you find out $f_n$ does not necessarily converge to $f$ uniformly. However the answer you provide assumes $f_n$ to converge uniformly so it is not answering this question.
$endgroup$
– Pedro Gomes
Jan 13 at 15:13




$begingroup$
@Jakobian My question is not a duplicate. If you read carefully the exercise you find out $f_n$ does not necessarily converge to $f$ uniformly. However the answer you provide assumes $f_n$ to converge uniformly so it is not answering this question.
$endgroup$
– Pedro Gomes
Jan 13 at 15:13












$begingroup$
@PedroGomes no, it doesn't. It's exactly the answer to your question
$endgroup$
– Jakobian
Jan 13 at 15:20






$begingroup$
@PedroGomes no, it doesn't. It's exactly the answer to your question
$endgroup$
– Jakobian
Jan 13 at 15:20














$begingroup$
@DavidC.Ullrich In order to apply Ergoroff I need to prove the measure of the domain where the function converges is finite. It is already assumed in the question when $f_nto f$ a.e that $mu({x:g(x)>lambda})=0$.
$endgroup$
– Pedro Gomes
Jan 13 at 15:21






$begingroup$
@DavidC.Ullrich In order to apply Ergoroff I need to prove the measure of the domain where the function converges is finite. It is already assumed in the question when $f_nto f$ a.e that $mu({x:g(x)>lambda})=0$.
$endgroup$
– Pedro Gomes
Jan 13 at 15:21






1




1




$begingroup$
How do I know that you are pointing me in the right direction if you do not prove you are? Does it not sound like an authority fallacy?
$endgroup$
– Pedro Gomes
Jan 13 at 15:32




$begingroup$
How do I know that you are pointing me in the right direction if you do not prove you are? Does it not sound like an authority fallacy?
$endgroup$
– Pedro Gomes
Jan 13 at 15:32










1 Answer
1






active

oldest

votes


















0












$begingroup$

Ok, a bigger hint. Let $$E_k={x:g(x)>1/k}.$$Since $mu(E_k)<infty$, Egoroff shows that there exists $S_ksubset E_k$ such that $f_nto f$ uniformly on $E_ksetminus S_k$ and $$mu(S_k)<epsilon/2^k.$$



So if $S=bigcup_{k=1}^infty S_k$ then $mu(S)<epsilon$. And it's possible to prove that $f_nto f$ uniformly on $Xsetminus S$. (There's still something to be proved in that last sentence, it's not quite just trivial by definition. Hint: So far we haven't used the fact that $|f_n|le g$.)






share|cite|improve this answer









$endgroup$













  • $begingroup$
    Is it not $E_k={x:g(x)<1/k}$. $S_k$ is subset of the set where $f_n$ converges to $f$? Or is there something I am missing?
    $endgroup$
    – Pedro Gomes
    Jan 13 at 17:50










  • $begingroup$
    $E_k$ is not what you say it is. So I don't know whether you're asking about $E_k$ or about ${x:g(x)<1/k}$. But in either case: So what?
    $endgroup$
    – David C. Ullrich
    Jan 13 at 23:09


















1 Answer
1






active

oldest

votes








1 Answer
1






active

oldest

votes









active

oldest

votes






active

oldest

votes









0












$begingroup$

Ok, a bigger hint. Let $$E_k={x:g(x)>1/k}.$$Since $mu(E_k)<infty$, Egoroff shows that there exists $S_ksubset E_k$ such that $f_nto f$ uniformly on $E_ksetminus S_k$ and $$mu(S_k)<epsilon/2^k.$$



So if $S=bigcup_{k=1}^infty S_k$ then $mu(S)<epsilon$. And it's possible to prove that $f_nto f$ uniformly on $Xsetminus S$. (There's still something to be proved in that last sentence, it's not quite just trivial by definition. Hint: So far we haven't used the fact that $|f_n|le g$.)






share|cite|improve this answer









$endgroup$













  • $begingroup$
    Is it not $E_k={x:g(x)<1/k}$. $S_k$ is subset of the set where $f_n$ converges to $f$? Or is there something I am missing?
    $endgroup$
    – Pedro Gomes
    Jan 13 at 17:50










  • $begingroup$
    $E_k$ is not what you say it is. So I don't know whether you're asking about $E_k$ or about ${x:g(x)<1/k}$. But in either case: So what?
    $endgroup$
    – David C. Ullrich
    Jan 13 at 23:09
















0












$begingroup$

Ok, a bigger hint. Let $$E_k={x:g(x)>1/k}.$$Since $mu(E_k)<infty$, Egoroff shows that there exists $S_ksubset E_k$ such that $f_nto f$ uniformly on $E_ksetminus S_k$ and $$mu(S_k)<epsilon/2^k.$$



So if $S=bigcup_{k=1}^infty S_k$ then $mu(S)<epsilon$. And it's possible to prove that $f_nto f$ uniformly on $Xsetminus S$. (There's still something to be proved in that last sentence, it's not quite just trivial by definition. Hint: So far we haven't used the fact that $|f_n|le g$.)






share|cite|improve this answer









$endgroup$













  • $begingroup$
    Is it not $E_k={x:g(x)<1/k}$. $S_k$ is subset of the set where $f_n$ converges to $f$? Or is there something I am missing?
    $endgroup$
    – Pedro Gomes
    Jan 13 at 17:50










  • $begingroup$
    $E_k$ is not what you say it is. So I don't know whether you're asking about $E_k$ or about ${x:g(x)<1/k}$. But in either case: So what?
    $endgroup$
    – David C. Ullrich
    Jan 13 at 23:09














0












0








0





$begingroup$

Ok, a bigger hint. Let $$E_k={x:g(x)>1/k}.$$Since $mu(E_k)<infty$, Egoroff shows that there exists $S_ksubset E_k$ such that $f_nto f$ uniformly on $E_ksetminus S_k$ and $$mu(S_k)<epsilon/2^k.$$



So if $S=bigcup_{k=1}^infty S_k$ then $mu(S)<epsilon$. And it's possible to prove that $f_nto f$ uniformly on $Xsetminus S$. (There's still something to be proved in that last sentence, it's not quite just trivial by definition. Hint: So far we haven't used the fact that $|f_n|le g$.)






share|cite|improve this answer









$endgroup$



Ok, a bigger hint. Let $$E_k={x:g(x)>1/k}.$$Since $mu(E_k)<infty$, Egoroff shows that there exists $S_ksubset E_k$ such that $f_nto f$ uniformly on $E_ksetminus S_k$ and $$mu(S_k)<epsilon/2^k.$$



So if $S=bigcup_{k=1}^infty S_k$ then $mu(S)<epsilon$. And it's possible to prove that $f_nto f$ uniformly on $Xsetminus S$. (There's still something to be proved in that last sentence, it's not quite just trivial by definition. Hint: So far we haven't used the fact that $|f_n|le g$.)







share|cite|improve this answer












share|cite|improve this answer



share|cite|improve this answer










answered Jan 13 at 16:27









David C. UllrichDavid C. Ullrich

60.9k43994




60.9k43994












  • $begingroup$
    Is it not $E_k={x:g(x)<1/k}$. $S_k$ is subset of the set where $f_n$ converges to $f$? Or is there something I am missing?
    $endgroup$
    – Pedro Gomes
    Jan 13 at 17:50










  • $begingroup$
    $E_k$ is not what you say it is. So I don't know whether you're asking about $E_k$ or about ${x:g(x)<1/k}$. But in either case: So what?
    $endgroup$
    – David C. Ullrich
    Jan 13 at 23:09


















  • $begingroup$
    Is it not $E_k={x:g(x)<1/k}$. $S_k$ is subset of the set where $f_n$ converges to $f$? Or is there something I am missing?
    $endgroup$
    – Pedro Gomes
    Jan 13 at 17:50










  • $begingroup$
    $E_k$ is not what you say it is. So I don't know whether you're asking about $E_k$ or about ${x:g(x)<1/k}$. But in either case: So what?
    $endgroup$
    – David C. Ullrich
    Jan 13 at 23:09
















$begingroup$
Is it not $E_k={x:g(x)<1/k}$. $S_k$ is subset of the set where $f_n$ converges to $f$? Or is there something I am missing?
$endgroup$
– Pedro Gomes
Jan 13 at 17:50




$begingroup$
Is it not $E_k={x:g(x)<1/k}$. $S_k$ is subset of the set where $f_n$ converges to $f$? Or is there something I am missing?
$endgroup$
– Pedro Gomes
Jan 13 at 17:50












$begingroup$
$E_k$ is not what you say it is. So I don't know whether you're asking about $E_k$ or about ${x:g(x)<1/k}$. But in either case: So what?
$endgroup$
– David C. Ullrich
Jan 13 at 23:09




$begingroup$
$E_k$ is not what you say it is. So I don't know whether you're asking about $E_k$ or about ${x:g(x)<1/k}$. But in either case: So what?
$endgroup$
– David C. Ullrich
Jan 13 at 23:09



Popular posts from this blog

Can a sorcerer learn a 5th-level spell early by creating spell slots using the Font of Magic feature?

ts Property 'filter' does not exist on type '{}'

Notepad++ export/extract a list of installed plugins